LSAT and Law School Admissions Forum

Get expert LSAT preparation and law school admissions advice from PowerScore Test Preparation.

 SherryZ
  • Posts: 124
  • Joined: Oct 06, 2013
|
#12011
HI Dave,

Thank you for your reply!

I have another question about this game:

In the last condition, it says "O is numbered higher than EITHER of K". I wanna know whether it means "O is numbered higher than BOTH K", or it means "O is numbered higher than ONE OF K"?? Sry, I am not English speaker :cry:

Thanks again!

---Sherry
User avatar
 Dave Killoran
PowerScore Staff
  • PowerScore Staff
  • Posts: 5850
  • Joined: Mar 25, 2011
|
#12012
Hi Sherry,

The "than either" construction is actually explained here on the forum, in some detail. It's a very tricky point:

http://forum.powerscore.com/lsat/viewto ... ?f=7&t=864

and http://forum.powerscore.com/lsat/viewto ... f=7&t=2615

Please let me know if those help. Fortunately, this "than either" issue hasn't appeared very often.

Thanks!
 srcline@noctrl.edu
  • Posts: 243
  • Joined: Oct 16, 2015
|
#22378
Hello

My problem with this game is 1. the relationship between rules 5 and 7, and 2. how to diagram rule 7. I know this rule was explained in previous posts, but I am still having a hard time understanding this.

I initially had this diagrammed as :
L>K>O (laria's aisle is an end aisle)

The fifth rule was diagrammed as
MKM which I understand.

I know Dave explained the templates in post 3 but I'm still pretty confused. Is this a proper inference: L> MKM (block)> O

Please help
Sarah
 Nikki Siclunov
PowerScore Staff
  • PowerScore Staff
  • Posts: 1362
  • Joined: Aug 02, 2011
|
#22399
Hi Sarah,

Assuming the isle numbers increase to the right, the fifth rule produces the following block:

MKM

The last rule states that O is numbered higher than either of K, and lower than at least one L. The rule has two components:

1. At least one L must be higher than O:

O - L

2. Both K's must be lower than O:

K - K - O

When combined, the two parts of the rule produce the following diagram:

K - K - O - L

Since K stocks isle 2 but M does not stock isle 1, the K inside the MKM block cannot be the K that stocks isle 2. So, we have:

K - MKM - O - L

Whereby the first K occupies isle 2.

Hope this helps!

Thanks,
User avatar
 ivan.l99
  • Posts: 16
  • Joined: Sep 16, 2023
|
#103677
For template #2, why can't L/J go on 7 or 8, instead O? Or am I misreading it
 Luke Haqq
PowerScore Staff
  • PowerScore Staff
  • Posts: 722
  • Joined: Apr 26, 2012
|
#103766
Hi Ivan.I99!

In all three templates, we know that L and J are going to stock aisles 1 and 9. One of the blocks that moves around in the templates is the MKM block.

In the second template, O has to be in either 7 or 8 because of the last rule ("Olga's aisle is numbered higher than either of Kurt's aisles..."). 7 or 8 are the lowest numbered aisles that O can stock. And O can't stock the last aisle, since O's aisle number must be lower than at least one of L's aisles.

But, to your question, it's possible that one or the other of spots 7 or 8 could be either L or J. It would just be necessary to ensure that no rules are broken by doing so (J can't occur consecutively, for example, and at O's aisle must be lower numbered than at least one of L's aisles).

Get the most out of your LSAT Prep Plus subscription.

Analyze and track your performance with our Testing and Analytics Package.